Mt 3 Soruları Flashcards

You may prefer our related Brainscape-certified flashcards:
1
Q

Clostridium türü bakterilerin çoğunlukla da Clostridium perfringens’in etken olduğu bu hastalık, travma veya cerrahi girişimler sonucu iskelet kaslarında ortaya çıkan nekrozla karakterize, progresif ve toksemik bir hastalıktır
Bu hangi hastalığı anlatmaktadır¿

A

Gazlı gangren (myonecrosis)

How well did you know this?
1
Not at all
2
3
4
5
Perfectly
2
Q

erythematous papules and plaques accompany itching and diminish within a day, which disease is this

A

Urticaria

rash of urticaria consists of circumscribed, raised, blanching erythematous papules and plaques, some of which have a central pallor, located on neck, arms and back. The mucous membranes, palms, and soles are spared

How well did you know this?
1
Not at all
2
3
4
5
Perfectly
3
Q

Which of the following pharmaceutical system is not used to deliver agents to treat dermatological disorders?

Emulsions
Cream
Transdermal therapeutic system
Oinment

A

Tts. It is for vitamins and painkillers

How well did you know this?
1
Not at all
2
3
4
5
Perfectly
4
Q

What is the most reliable test for the diagnosis of septic arthritis?

Blood culture
Procalcitonin
C reactive protein (CRP)
Synovial fluid aspirate
Erythrocyte sedimentation rate (ESR)

A

Synovial fluid aspirate

How well did you know this?
1
Not at all
2
3
4
5
Perfectly
5
Q

Which of the following is a deformity that does not require treatment and possibly resolves spontaneously?

Developmental dysplasia of the hip
Flexible pes planus
Tarsal coalitions Congenital
convex pes valgus /vertical talus
Pes Equino varus

A

Flexible pes planus

How well did you know this?
1
Not at all
2
3
4
5
Perfectly
6
Q

In which disease Pastia’s lines are seen?

Scarlet fever
Toxic epidermal necrolysis
Staphylococcus scalded skin syndrome
Recurrent perineal erythema
Streptococcal toxic shock syndrome

A

Scarlet fever

How well did you know this?
1
Not at all
2
3
4
5
Perfectly
7
Q

Which of the following physical examination method for diagnosis of developmental dysplasia of the hip is diagnostic?

Ortholani / Barlow
Swelling of the hip
Length discrepancy of the lower extremity
Limitation of abduction of hip
Pain on hip movement

A

Ortholani / Barlow

How well did you know this?
1
Not at all
2
3
4
5
Perfectly
8
Q

Which of the followings is not a component of compartment syndrome?

Paresthesia
Pulselessness
Plasmapheresis
Pain

A

Plasmapheresis.

Compartment syndrome is defined as elevation of pressure in a compartment. 5P’s of Compartment syndrome are pain-paresthesia- pulselessness-pallor and paralysis
It is a real emergency situation and should be treated with emergent fasciotomy, if diagnosed too late has devastating complications resulting with severe disabilities in the affected extremity.

Paresthesia: uyuşukluk karıncalanma hissi

How well did you know this?
1
Not at all
2
3
4
5
Perfectly
9
Q

Patient has mild fever for the last 3 days. Afterwards, her parents noticed a rash which was prominent on her face and trunk initially, then spreading to arms and legs. A few children in day-care had a similiar rash recently. Physical examination showed widespread presence of macules, papules, vesicles, and pustules in close proximity over her body. She was restless and itching her skin all the time during the examination

What is the diagnosis¿

Varicella
Measless
Mumps
Rubella

A

Varicella

How well did you know this?
1
Not at all
2
3
4
5
Perfectly
10
Q

INFO CARD
Plain film is the first imaging technique.
Osteid osteoms are typical mostly painful lesions.
Usg is the frist choice about new born congenital hip displasia.

A

Info

How well did you know this?
1
Not at all
2
3
4
5
Perfectly
11
Q

Which of the following statements regarding hip dislocation is not correct ?

The classic position of an anteriorly dislocated hip is flexion, abduction and external rotation.

90% of all pure dislocations are anterior hip dislocations.

Treatment involves closed reduction under anesthesia in order to avoid femoral or acetabular fractures.

Hip joint dislocation is the third most common dislocation seen in humans.

Following reduction, if the joint is concentric and stable; full weightbearing as tolerated is permitted.

A

90lı şık. Posterior doğrusu.

How well did you know this?
1
Not at all
2
3
4
5
Perfectly
12
Q

A 26-year-old man presents to the dermatologist complaining about the lesions that he has noticed recently on the skin adjacent to his penis. He is concerned about having acquired a sexually transmitted disease through one of several recent sexual encounters. The dermatologist defined the lesions as smooth dome- shaped/umblicated papules with a central depression. An excisonal biopsy is performed for one of the lesions. The histopathologic examination revealed verrucous epidermal hyperplasia and intracytoplasmic inclusions located to the corneal and granular layer of the epidermis.
Which of the following viruses can be the etiologic agent for this lesion?

Human papiloma virus
HIV
Hepatit B virus
Pox virus
Herpes simplex virus type 2

A

Pox.

The gross appearance and histologic appearance are characteristic of molluscum contagiosum, which is caused by poxvirus. The cells in the epidermis contain large intracytoplasmic inclusions, which are called molluscum bodies. The molluscum bodies are accumulations of viral particles.

Molluscum contagiosum, ciltte küçük pembe veya ten rengi kabarıklıklara neden olan virüs kaynaklı bir hastalıkdır. Zararlı değildir ve hastaların genellikle başka şikayetleri yoktur. Virüs şişliklerin içindedir ve hafif bulaşma riski bulundurmaktadır. Bu çıkıntılar genellikle uzun bir süre sonunda kaybolur.

How well did you know this?
1
Not at all
2
3
4
5
Perfectly
13
Q

Is a complete cord hemitransection usually seen after penetrating traumas of the spinal cord. Characterized by ipsilateral motor, proprioception, vibration and contralateral pain-temperature deficit. Has excellent prognosis depending on the cause.”
Which spinal cord injury pattern is defined above?

Brown-Sequard syndrome
Conus medullaris syndrome
Posterior cord syndrome
Central cord syndrome

A

Brown-Sequard syndrome

How well did you know this?
1
Not at all
2
3
4
5
Perfectly
14
Q

Info

The term melanocytic lesion refers to proliferations of neural crest-derived melanocytic cells in the skin ranging from benign freckles and nevi (moles) to malignant melanoma.

A

Info

How well did you know this?
1
Not at all
2
3
4
5
Perfectly
15
Q

Which of the following disease-modifying anti-rheumatic drugs (DMARDs) is the first and only drug to be indicated to slow down structural joint damage in romatoid arthritis (RA)?

Methotrexate
Sulfasalazine
Leflunomide
Hydroxychloroquine

A

Leflunomide is an immunomodulatory drug that decreases pyrimidine synthesis by inhibiting the enzyme “dihydroorotate dehydrogenase” (DHODH), inhibiting both T cell proliferation and production of auto-antibodies by B cells.

How well did you know this?
1
Not at all
2
3
4
5
Perfectly
16
Q

A 30-year-old man is evaluated for an intensely pruritic erythematous papular and vesicular dermatitis involving the forearms and buttocks bilaterally. He is treated with a high-potency topical steroid, which helps with the itching; however, the rash fails to resolve. After that a skin biopsy is performed from this patient, which shows granular deposits of IgA within the dermal papillae with direct immunofluorescence technique. Which of the following histopathological findings is related with this patient’s disease?

Suprabasal and intraepidermal clefting, extensive acantholysis of keratinocytes resembling dilapidated brick wall

Subepidermal blisters with papillary neutrophilic microabscesses

Suprabasal acantholytic blisters leaving a single layer of basal cells

Epidermal hyperplasia and severe spongiosis

Subepidermal non-acantholytic blisters and superficial perivascular inflammatory infiltrate of lymphocytes and eosinophils

A

Subepidermal blisters with papillary neutrophilic microabscesses

The distribution of the rash, the clinical description, and histopathological ad DIF findings (Subepidermal blisters with papillary neutrophilic microabscesses and granular deposits of IgA within the dermal papillae) all indicate dermatitis herpetiformis due to underlying celiac disease. The gluten-containing foods would be most likely to aggravate the patient’s condition. Dermatitis herpetiformis is associated with IgA autoantibodies to fibrils that bind the epidermal basement membrane to the dermis, and also produces subepidermal blisters. The association of dermatitis herpetiformis with celiac disease provides a clue to its pathogenesis. Genetically predisposed individuals develop lgA antibodies to dietary gluten (derived from the wheat protein gliadin).

Suprabasal acantholytic blisters leaving a single layer of basal cells resembling row of tombstones: Pemphigus vulgaris

Suprabasal and intraepidermal clefting, extensive acantholysis of keratinocytes resembling dilapidated brick wall: Hailey-hailey disease

Subepidermal non-acantholytic blisters and superficial perivascular mixed inflammatory infiltrate: Bullous pemphigoid

Epidermal hyperplasia and severe spongiosis can be the histopathological features of acute eczematous dermatitis (spongiotic dermatitis), not a bullous disease

How well did you know this?
1
Not at all
2
3
4
5
Perfectly
17
Q

Which of the following is not associated with the prolonged course of urticaria?

Presence of IgG autoantibodies
Initial disease severity
Diurnal variation
Presence of angioedema
Combination of CSU (Chronic spontaneous urticaria) and physical urticarias

A

Diurnal variation

How well did you know this?
1
Not at all
2
3
4
5
Perfectly
18
Q

Which of the following is not a feature of inflammatory arthritis?

Erythema
Symmetric pain even at rest
Joint swelling
Warmth
Typically about 10 minutes of morning stiffness

A

Typically about 10 minutes of morning stiffness

Genelde inflammatory artrit tipi olan RA dan görülebileceği üzere 10dktan uzun sürer. RA 30dkdan uzun sürer mesela

How well did you know this?
1
Not at all
2
3
4
5
Perfectly
19
Q

Which bactery cause cellulitis¿

A

S pyogenes

How well did you know this?
1
Not at all
2
3
4
5
Perfectly
20
Q

Which of the followings are the signs / symptoms of osteoarthritis?

Locking sensation of the knee
Crepitations
Pain
Morning stiffness lasting for more than an hour
Gait changes

A

Morning stiffness içeren şık dışında hepsi. Çünkü 30dkdan az sürer

How well did you know this?
1
Not at all
2
3
4
5
Perfectly
21
Q

Which of the following sentences is not correct for cellulitis?

Group A Streptoccoccal and Staphylococcal species are the main causative agents.
Cellulitis is a common infection of the deep dermis and subcutaneous tissue.
Children are more likely than adults to have facial involvement.
Unilateral lower extremity involvement is typical.
Cellulitis has well defined borders.

A

Cellulitis DOES NOT have well defined borders. Others are correct.

How well did you know this?
1
Not at all
2
3
4
5
Perfectly
22
Q

What is the most common agent that causes septic arthritis

A

S aureus

How well did you know this?
1
Not at all
2
3
4
5
Perfectly
23
Q

A 53-year-old woman presents with a two-month history of difficulty climbing stairs and arising from the seated position. On physical examination, she has a purplish discoloration of the skin over the forehead, eyelids, and cheeks. She has tenderness on palpation of the quadriceps muscles.
What’s the most likely diagnosis?

Rheumatoid arthritis
Granulomatosis with polyangiitis
Systemic sclerosis
Dermatomyositis
Felty’s syndrome

A

Dermatomyositis

Dermatomiyozit nadir görülen bir inflamatuar hastalıktır. Bu rahatsızlığın gelişmesi ile birlikte ciltte döküntü, kas güçsüzlüğü ve inflamatuar miyopati olarak bilinen kas iltihaplanması şikayetleri ortaya çıkar. Dermatomiyozit her yaştan bireyi etkileyebilen bir durumdur.

Granulomatosis with polyangiitis (GPA) is a rare vasculitis affecting small vessels. Hallmark features include necrotizing granulomas and pauci-immune vasculitis that most commonly affects the upper respiratory tract, lungs, and kidneys.

Felty syndrome is a rare extra-articular manifestation of seropositive rheumatoid arthritis characterized by RA, neutropenia, and splenomegaly. While the etiology remains somewhat obscure, baseline data suggests an association with certain human leukocyte antigen (HLA) subtypes.

How well did you know this?
1
Not at all
2
3
4
5
Perfectly
24
Q

loss of joint space, subchondral sclerosis (hardening of the bone just beneath the cartilage), and calcium deposition within the surrounding tissues typically indicates which disease¿

A

loss of joint space, subchondral sclerosis (hardening of the bone just beneath the cartilage), and calcium deposition within the surrounding tissues typically indicates osteoarthritis. Osteoarthritis is a degenerative joint disease characterized by the breakdown of joint cartilage and underlying bone changes. Calcium deposition, also known as calcification or calcinosis, can occur in the soft tissues surrounding the joint as a result of the degenerative process.

How well did you know this?
1
Not at all
2
3
4
5
Perfectly
25
Q

Which of the following is incorrect regarding fungal infections of skin and soft tissue?

Typical ringworm lesions have an inflamed circular border containing
papules and vesicles surrounding a clear area of relatively normal
skin.

Dermatophytes infect only deeper tissues not superficial keratinized tissues.

The most commonly used medium for cultivating fungi is Sabouraud’s
agar, which contains only glucose and peptones as nutrients.

The simplest method is to mix a clinical specimen, such as skin
scrapings, with a 10% solution of potassium hydroxide (KOH) on a
microscope slide under a coverslip.

Fungi can often be identified by directly observing their distinctive
morphologic features on direct microscopic examination of infected
pus, fluids, or tissues.

A

Dermatophytes infect only superficial keratinized tissues therefore 2nd option is the answer

How well did you know this?
1
Not at all
2
3
4
5
Perfectly
26
Q

A 36-year-old man presented with the complaint of the presence of asymptomatic recurrent scales around the nose and on the eyebrows. Physical examination revealed greasy yellowish scales on mild erythematous ground in these areas. The patient was otherwise completly healthy.
What is the most likely diagnosis in this patient?

Discoid eczema
Allergic contact dermatitis
Psoriasis vulgaris
Atopic dermatitis
Seborrhoeic dermatitis

A

Seborrhoeic dermatitis

Seborrhoeic dermatitis usually affects the seborrhoeic and hairy areas such as scalp, ears, eyebrows, face, pre-sternal area, armpits, umblicus and groin. It shows a chronic and recurrent course, and characterized by greasy yellowish scales

Türkçesi egzama

How well did you know this?
1
Not at all
2
3
4
5
Perfectly
27
Q

Which of the following features regarding discoid lupus erythematosus are correct?

1 Linear granular deposition of immunoglobulin A along the
dermoepidermal and dermal follicular junctions

2 Characterized by a lichenoid infiltrate of lymphocytes along the dermoepidermal junction

3 A typical sign of red butterfly-shaped rash over the cheeks and nose sparing the nasolabial folds occur

4 Systemic manifestations of systemic lupus erythematosus are also
common in this disease

5 Most common chronic form of cutaneous lupus

A

1 ve 4 yanlış.

The features of DLE: -Systemic manifestations of systemic lupus erythematosus are mostly absent / rare in this disease -A typical sign of red butterfly-shaped rash over the cheeks and nose sparing the nasolabial folds occur
-Histopathologically characterized by a lichenoid infiltrate of lymphocytes along the dermoepidermal junction and/or dermal- follicular epithelial interface -Linear granular deposition of immunoglobulin G and complement along the dermoepidermal and dermal follicular junctions

How well did you know this?
1
Not at all
2
3
4
5
Perfectly
28
Q

Does osteophytes seen in OA or RA

A

OA

Bone spurs, or osteophytes, are bony growths that form in your joints or in the spine. They cause damage to your bones, muscles, or tendons, often as a result of osteoarthritis. These smooth growths may not cause any symptoms or need treatment.

How well did you know this?
1
Not at all
2
3
4
5
Perfectly
29
Q

Is pannus formation seen in OA or RA

A

RA

Normalde hücre ve damar yapısından fakir olan sinoviyum, kanlanmanın fazla olduğu, proliferasyon gösteren, tümör dokusuna benzer destrüktif bir doku haline gelir. Bu dokuya PANNUS denir.

How well did you know this?
1
Not at all
2
3
4
5
Perfectly
30
Q

Are bouchard and heberden nodes seen in Oa or RA

A

Oa

Heberden distal bouchrad meta parmak eklemlerinde oluşan şişkinlikler

How well did you know this?
1
Not at all
2
3
4
5
Perfectly
31
Q

Morning stiffnes that gets improved by activity s seen in OA OR RA

A

OA

How well did you know this?
1
Not at all
2
3
4
5
Perfectly
32
Q

Inhibition of which of the following enzymes most likely mediated the therapeutic effect of leflunomide added to methotrexate therapy in a 40-y.o. woman suffering from rheumatoid arthritis?

Dihydrofolate reductase
Type II topoisomerase
Cyclooxygenase-1
Dihydroorotate dehydrogenase

A

Dihydroorotate dehydrogenase

Leflunomide is a disease-modifying antirheumatic drug (DMARD)
immunosuppressant approved for the treatment of rheumatoid and psoriatic arthritis. Its mechanism of action involves the inhibition of dihydroorotate dehydrogenase, an enzyme involved in pyrimidine synthesis. As a consequence, there is a reduction in uridine triphosphate levels and pyrimidine synthesis in leucocytes and other rapidly dividing cells. Leflunomide is about as effective as methotrexate in rheumatoid arthritis and enhances methotrexate activity when given concomitantly. Therefore, it is often added to methotrexate therapy, as in this case.

How well did you know this?
1
Not at all
2
3
4
5
Perfectly
33
Q

A specific chromosomal translocation, t(11;22) (q24;q12) or a variant is found in most of the said diseases family of tumors.

Clinical manifestations may include constitutional symptoms such as fever, fatigue, and weight loss.

Which tumor is this¿

A

Ewing sarcoma

How well did you know this?
1
Not at all
2
3
4
5
Perfectly
34
Q

The patient has enlarging lesion on his upper back. The lesion is an erythematous, tender, round, perifollicular nodule with multiple pustules protruding from the surface. S aureus is most common causative agent of the disease.

Which disease is this

A

Carboncule (çıban)

How well did you know this?
1
Not at all
2
3
4
5
Perfectly
35
Q

A previously healthy 18 month old infant developed high fever abruptly. His parents saw that the body temperature was 40 other finding and he seemed well when the fever was controlled with paracetamol. However, three days later, a rapid defervescence (abrupt loss) of fever followed by the onset of a pink rash over the trunk was noticed. You examined the baby. He was a well developed and healthy boy, having no fever, and except for the mild, pink, morbilliform rash, other physical findings were unremarkable.The clinical follow up showed that the rash resolved in two days.
What is your diagnosis?

Chickenpox
Roseola infantum
Erythema infectiosum
Rubella
Measles

A

Roseola infantum: 6 aylıktan daha küçük çocuklar anneden geçen antikorlar nedeniyle altıncı hastalığa karşı bağışıktırlar. Roseola infantum olarak da isimlendirilen altıncı hastalıkta oluşan en ayırt edici belirti rahatsızlığın 3-7. günleri arasında yüksek ateş sonrası gelişen cilt döküntüsüdür.

How well did you know this?
1
Not at all
2
3
4
5
Perfectly
36
Q

The wife of a 56-year-old man comments that the ‘mole’ on the back of her husband’s neck has seemed to be much larger than she remembered it 2 months ago. On physical examination it is 1 x 1.2 cm and has irregular pigmentation and irregular borders. The biopsy of this lesion seen here microscopically. Histopathologically the lesion is composed of large pigmented polygonal cells with large nucleoli and the lesion has infiltrative border.
Which of the following is the most likely diagnosis?

Acanthosis nigricans
Actinic keratosis
Dermatofibroma
Hypertrophic scar
Malignant Melanoma
Verruca vulgaris

A

Malignant melanoma.

Changes in ‘moles’ are suspicious for melanoma. Note the large polygonal cells with large nucleoli and cytoplasmic pigmentation. The features of the lesion is compatible with malignant melanoma

How well did you know this?
1
Not at all
2
3
4
5
Perfectly
37
Q

Verrucous epidermal hyperplasia with a cup-like architecture and large intracytoplasmic inclusions within epidermis

A

Molluscum contagiosum

How well did you know this?
1
Not at all
2
3
4
5
Perfectly
38
Q

Collection of neutrophils beneath the stratum corneum and formation of subcorneal pustules accompanied with bacteria

A

Impetigo

How well did you know this?
1
Not at all
2
3
4
5
Perfectly
39
Q

Epidermal hyperplasia, koilocytosis, keratohyalin granules and intracytoplasmic eosinophilic inclusions

A

Verrucae vulgaris

How well did you know this?
1
Not at all
2
3
4
5
Perfectly
40
Q

Hydropic swelling of epidermis, intraepithelial vesicular lesions, formation of pink to purple intranuclear inclusions containing virions, İnclusion-bearing multinucleated syncytia

A

Herpes simplex infection

How well did you know this?
1
Not at all
2
3
4
5
Perfectly
41
Q

Fungal microorganisms in hyphae and yeast forms (spaghetti and meatballs appearance) found within the cornified layer associated with neutrophilic infiltration

A

Tinea versicolor

How well did you know this?
1
Not at all
2
3
4
5
Perfectly
42
Q

Which eruptive disease of childhood shows the following characteristic eruption pattern?
Exanthema typically beginning at the hairline and spreading in a cranial to caudal direction; the lesions are red macules initially, then progress to papules, vesicles, and pustules and then crust within 48 hours. Different highly pruritic skin rashes (macules, papules, vesicles, pustules) present in the same anatomic area.

Chickenpox
Roseola infantum
Measles
Erythema infectiosum

A

Chickenpox

How well did you know this?
1
Not at all
2
3
4
5
Perfectly
43
Q

Which of the following is a purine analog and acts through inhibition of xanthine oxidase which is especially reserved for chronic treatment of severe tophaceous gout or renal urate stones?

Lesinurad
Febuxostat
Colchicine
Allopurinol

A

Allopurinol.

It is the only purine (note “purin” in its name) analog Feedback:
within antigout drugs acting through inhibition of xanthine oxidase reserved for chronic treatment of severe tophaceous gout or renal urate stones.

How well did you know this?
1
Not at all
2
3
4
5
Perfectly
44
Q

77-year-old woman with a sore, pruritic, blistering eruption for three days prior to arrival. Examination of the palms, soles, dorsal aspect of the feet, legs, buttocks, and inner thighs revealed well-demarcated, tense bullae (fig1). They ranged in size from 1cm to 3 cm. They were not clustered in a rosette pattern. There was oral mucosal involvement and also crusting over the left lower lip. On the histopathologic examination; surface epithelium cleanly detached from the underlying connective tissue at the point of the basement membrane zone is revealed (fig2). On direct immunofluorescence deposits of IgG along the basement membrane zone were detected in a linear pattern (fig3).
Which of the following is the most probable diagnosis of this patient?

Hailey hailey disease
Bullous pemphigoid
Pemphigus vulgaris
Linear IgA bullous disease
Dermatitis herpetiformis

A

Bullous pemphigoid: Chronic, immune mediated, subepithelial blistering disease that predominantly affects mucous membranes. Autoantibodies directed at components of the basement membrane zone (Autoantibodies are against to the components of the basement membrane, particularly the BP antigens BP180 and BP230). Lesions present as tense bullae which do not easily rupture because of the deep location. DIF reveals linear deposition of immunoglobulin and complement at DEJ

How well did you know this?
1
Not at all
2
3
4
5
Perfectly
45
Q

Joint infection may be caused by the spread of microorganisms from the near epiphysis. This route can not be used for a specific period of time due to the existence of a natural barrier of the physis plate.
Which of the following refers to that period of time?

Between 18 months and 16 years
0-18 months
0-6 months
>18 months to 24 months
Adulthood
In fetal period

A

Between 18 months and 16 years

The existence of transpyhseal vessels may induce a joint Feedback:
infection which may arise from epiphysis osteomyelitis. An open physis line is a natural barrier to resist such an infection. The existence of such a barrier usually blocks the involvement of joints from nearby epiphyseal/metaphyseal osteomyelitis.

How well did you know this?
1
Not at all
2
3
4
5
Perfectly
46
Q

Which of the following is the mechanism of action of a drug administered to a 50- y.o. man who developed muscle rigidity, tachycardia, labile blood pressure, profuse diaphoresis, and high fever (>40°C) shortly after the initiation of general anesthesia with halothane and succinylcholine prior to his inguinal hernia surgery? (The anesthesia was discontinued at once, and the drug of interest was administered by rapid intravenous push.)

Blockade of excitatory neurotransmitter release in the brain
Blockade of Ca2+ channels in the sarcoplasmic reticulum
Increased K + conductance in the skeletal muscle cell membrane
Blockade of Ca 2+ channels in the skeletal muscle cell membrane
Activation of GABA B receptors in the spinal cord

A

Blockade of Ca 2+ channels in the sarcoplasmic reticulum

The clinical picture of the patient suggests the diagnosis of Feedback:
malignant hyperthermia, a rare but potentially life-threatening disorder that can be triggered by a variety of stimuli, including the administration of certain anesthetics (mainly halothane) and neuromuscular blocking agents (mainly succinylcholine). The disorder is related to a hereditary impairment in the ability of the sarcoplasmic reticulum (SR) to sequester calcium (Ca 2+
);
therefore, a prolonged release of Ca 2+
occurs after the triggering
event through SR Ca 2+
channels (ryanodine receptors, RyR). This
causes massive muscle contraction, hyperthermia, and lactic acidosis. Dantrolene is a drug of choice in this disorder because it blocks RyRs in the skeletal muscle SR, thus preventing the massive release of Ca 2+
. Cardiac and smooth muscle are
minimally affected by dantrolene because they have a different subtype of RyRs in their SR.

How well did you know this?
1
Not at all
2
3
4
5
Perfectly
47
Q

Which of the following statements regarding a triad in a striated muscle are correct?

It is formed by two terminal cisternae and one t-tubule.
Propagates the action potential from the sarcolemma to the terminal cisternae.
It forms at the level of A-I junction of a sarcomere.
It is found in cardiac muscle.
It contains fascia adherens, gap junctions and desmosomes

A

1 2 3 doğru

How well did you know this?
1
Not at all
2
3
4
5
Perfectly
48
Q

Which of the following class of drugs is not indicated for eczema treatment?

Calcineurin inhibitors
Glucocorticoids
Antibacterials
Retinoids

A

Antibacterials

Eczema is not caused by bacteria, however, glucocorticoids may be combined with antibacterial or antifungal agents if infection is present.

How well did you know this?
1
Not at all
2
3
4
5
Perfectly
49
Q

A 35-year-old woman presents with pain, fever and swelling in her right leg. Examination reveals well-demarcated erythema and spreading streaks of lymphedema.
Which of the following is the most likely diagnosis of this patient’s condition?

Necrotising cellulitis
Myositis
Erysipelas
Necrotising fasciitis
Impetigo

A

Erysipelas

This patient has clinical features of erysipelas, a type of superficial cellulitis with dermal and superficial lymphatic involvement.
Erysipelas is a bacterial infection that leads to pain, edema, and well-demarcated erythema.

How well did you know this?
1
Not at all
2
3
4
5
Perfectly
50
Q

Which of the following features are considered as a risk factor in the development of Developmental dysplasia of the hip (DDH)?

Aged mother
Positive family history
Vertex presentation at birth
Male sex
Intrauterine packaging problems

A

2 ve 5

How well did you know this?
1
Not at all
2
3
4
5
Perfectly
51
Q

Which of the below listed joints are considered as a ball and socket type joint?

The subtalar joint
The knee joint
The hip joint
The metacarpophalangeal joints
The shoulder joint

A

Hip and shoulder

How well did you know this?
1
Not at all
2
3
4
5
Perfectly
52
Q

A 3 year boy was brought to the family health care center because of fever and rash. From the history you learned that he had fever, cough, a running nose and red eyes for the last couple of days, and yesterday morning the mother noticed a red rash on his face and neck , which is now spreading to his chest and back. Mother told that two other children at the same day care had similiar rash.
His physical examination was remarkable for fever, cough, coryza, conjuntivitis, and a discrete morbiliform rash in the form of coalescing erythematous macules and papules . What is your most likely diagnosis ?

Rubella
Erythema Infectiosum
Scarlet fever
Measles

A

Measles

These signs and symptoms strongly suggest measless. Feedback:
Particularly cough+coryza+conjuctivitis ( 3Cs) and morbiliform rash.

How well did you know this?
1
Not at all
2
3
4
5
Perfectly
53
Q

A 65-year-old woman presented with a 1 cm × 1 cm erythematous keratotic plaque on the palmar aspect of the left hand seen at the first picture. Physical examination was otherwise unremarkable. The lateral edge of the lesion was biopsied for further evaluation. Histopathological examination showed hyperkeratosis, parakeratosis, and full thickness atypia of the epidermis. No invasion beyond the basement membrane was noted.
According to the microscopic findings which of the following is the most likely diagnosis of this lesion?

Psoriasis
Seborrheic keratosis
Verruca vulgaris
Bowen’s disease
Squamous cell carcinoma

A

Bowen’s disease.

Bowen’s disease, or squamous cell carcinoma (SCC) in situ, is an intraepithelial neoplasm with a risk of progression to SCC. It is most often located in sun-exposed areas. Head and neck are most commonly involved followed by the lower and the upper extremities. While the dorsal aspects of the hands are commonly affected, occurrence on the palms is rare. Histopathologic exam showing full-thickness atypia of the epidermis with loss of normal maturation of the keratinocytes will differentiate palmar Bowen’s disease from the other entities.

How well did you know this?
1
Not at all
2
3
4
5
Perfectly
54
Q

Which of the following disease-modifying antirheumatic drugs (DMARDs) is a mouse/human chimeric monoclonal antibody against TNF alpha?

Abatacept
Infliximab
Adalimumab
Etanercept

A

Infliximab

Infliximab is a cytotoxic drug that inhibits the activity of the T-cells and B-cells. It is a mouse/human chimeric (denoted by “xi”) monoclonal antibody (mab) that binds to and inhibits the activity of tumor necrosis factor (TNF-alpha). TNF-alpha is a mediator of inflammation and immune activation. The main indication for infliximab is in the treatment of Crohn’s disease, which is an inflammatory condition of the intestinal tract.

How well did you know this?
1
Not at all
2
3
4
5
Perfectly
55
Q

A 4 year-old child is referred to pediatric clinic with limping. He had an upper respiratory tract infection prior to his complaints. He did not have high fever and any rash. On physical exam he looks well, there is painful hip on flexion. There is no hepatosplenomegaly. Complete blood count including acute phase reactants and antistreptolysin antibodies are normal. What is the provisional diagnosis?

Brucella arthritis
Perthes disease
Toxic synovitis
Septic arthritis
Acute rheumatic fever

A

Toxic synovitis

How well did you know this?
1
Not at all
2
3
4
5
Perfectly
56
Q

Which of the following statements are correct for hip dislocations? (Choose as many as required)

The main treatment of hip dislocations is urgent open (surgical) reduction and fixation of the hip joint with bone screws.

It is the most common joint dislocation seen in humans.

Posterior dislocation is the most commonly seen type among all hip dislocations

In posterior hip dislocation, the leg on the affected side lies in flexed, internally rotated and adducted position.

The ethiology is traumatic in the majority of cases.

A

3 4 ve 5 doğru.

The most common joint dislocation seen in humans is shoulder dislocation, followed by elbow and hip dislocation. The ethiology is traumatic in the majority of cases. Posterior dislocation is the most commonly seen type among all hip dislocations, which the leg on the affected side lies in flexed, internally rotated and adducted position. The main treatment of hip dislocations is urgent closed reducton. Surgical reduction is preferred on late admittances and failed close reductions.

How well did you know this?
1
Not at all
2
3
4
5
Perfectly
57
Q

Which of the following tumor reveals a histology similar to lipoma which is composed of a mixture of normal-appearing adipocytes intermixed with atypical adipocytes?

Dedifferentiated liposarcoma
Well-differentiated liposarcoma
Rhabdomyosarcoma
Myxoid liposarcoma
Pleomorphic liposarcoma

A

Well-differentiated liposarcoma

Histology of well-differentiated liposarcoma is similar to lipoma Feedback:
showing a mixture of normal-appearing adipocytes intermixed with atypical adipocytes. Pleomorphic liposarcoma histology differs with pleomorphic cells, myxoid liposarcoma differs with myxoid background. Dedifferentiated liposarcoma is hard to diagnose by just adipocytes and rhabdomyosarcoma is a skeletal muscle tumor.

How well did you know this?
1
Not at all
2
3
4
5
Perfectly
58
Q

Which of the listed factors are considered as risk factors for developmental dysplasia of hip?

Firstborn
Male sex
Breech presentation at birth
Oligohydramnios
Old mothers

A

1 3 4

Firstborn (primiparity) babies, female sex, breech presentation at Feedback:
birth, young mothers, positive family history, intrauterine packaging problems, oligohydramnios, newborns with greater weight and height, newborns with deformities of the feet or spine are considered as risk factors for DDH.

How well did you know this?
1
Not at all
2
3
4
5
Perfectly
59
Q

Which one of the below listed reflexes is indicative of the termination of the spinal shock?

Spurling’s reflex
Achilles reflex
Bulbocavernosus reflex
Ankle clonus
Babinsky reflex

A

Bulbocavernosus reflex

Bulbocavernous reflex is the first reflex to return once the spinal Feedback:
shock is over; it’s presence heralds the end of the stage of spinal shock and onset of recovery.

How well did you know this?
1
Not at all
2
3
4
5
Perfectly
60
Q

Which one of the below listed tests is used for examining lumbar spinal mobility?

Hoffmann test
Laseque test
Schober’s test
Patrick’s test

A

Schober’s test

How well did you know this?
1
Not at all
2
3
4
5
Perfectly
61
Q

Which one of the listed factors is not considered as a treatment goal in the treatment of Legg Perthes Calve disease?

Increasing the blood flow to the necrotic femoral epiphysis
Limitation of loading in order to prevent collapse of the head
Resolution of symptoms
Restoration of range of motion
Keeping the femoral head contained within the acetabulum

A

Increasing the blood flow to the necrotic femoral epiphysis

Conservative or surgical treatment options for LPC disease all aim to decrease pain, increase the ROM of the hip joint, keep the femoral head within the acetabulum, and prevent further collapse of the femoral head. Increasing the blood flow to the affected femoral head is not possible with the current treatment options.

62
Q

Which one of the statements listed below is not correct considering femoral shaft fractures?

Stress ractures may be seen in prolonged use of biphosphonates.
Rigid intramedullary nailing is the most preferred way of fixing femoral shaft fractures in the adult population.
Low impact injuries in the elderly may cause a femoral shat fracture due to osteoporosis.
Hip spica casting is the standard treatment of femoral fractures in the elderly.
In contaminated, open femoral shaft fractures, external fixation may temporarily be used in order to avoid complications.

A

Hip spica casting is the standard treatment of femoral fractures in the elderly.

Hip spica casting is the preferred treatment modality for femoral shaft fracture in children up to 5 years of age. Rigid fixation (intramedullary nailing or plating ) is the proper treatment in adults.

63
Q

A 50-year-old woman recently diagnosed with gouty arthritis started a treatment with a drug that inhibits leukocyte migration and phagocytosis secondary to inhibit ion of tubulin polymerization.
Which of the following drugs did the patient most likely take?

Colchicine
Indomethacin
Prednisone
Allopurinol

A

Colchicine can be used to stop an acute gout attack or, most often, for the prevent ion of further at tacks, as in this case. The drug binds to the intracellular protein, tubulin, thereby preventing its polymerization into microtubules and thus blocking mitosis in metaphase. Cells with the highest rate of division are affected early. Granulocyte migration into the inflamed area and phagocytosis of urate crystals by macrophages are inhibited, thus relieving the pain and inflammation of gouty arthritis. These actions are specific, and the drug is devoid of general analgesic or anti-inflammatory effects.

64
Q

Which of the following endogenous molecule is targeted by infliximab which was added to the ongoing treatment of a 50-year-old woman with refractory rheumatoid arthritis (RA)?

Interleukin-1 (IL-1)
Vascular endothelial growth factor (VEGF)
Interleukin-10 (IL-10)
Tumor necrosis factor-α (TNFα)

A

Tumor necrosis factor-α (TNFα)

Infiximab is a chimeric monoclonal antibody (mab) that binds with Feedback:
high affinity to soluble and possibly membrane-bound tumor necrosis factor-α (TNF-α). TNF-α is a proinflammatory cytokine that appears to be especially important in the inflammatory processes associated with autoimmune disorders, such as RA, ankylosing spondylitis, Crohn disease, and psoriasis. TNF-α-blocking agents such as infliximab are used in RA, especially for refractory type.

65
Q

A 10-year-old male presents with rashes. His condition started 5 days ago with fever, sore throat, and painful swallowing and then after 3 days, it was noticed that flushed cheeks and papular rashes on his trunk which are non-confluent lesions then it spread to the extremities. The rashes felt like sandpaper. The vital signs show a temperature of 39 C , cardiac rate 110/min, and a respiratory rate of 22 breaths/min. The physical exam shows blanching papular rashes on his trunk and extremities, tonsillopharyngeal congestion with exudate, and cervical lymphadenopathy.
Which one is not associated with this clinic?

Palmolpantar desquamation occurs in 7-10 days
Penicillines are used for the treatment
Glomerulonephritis can be seen as a complication
It is a toxin related disease
There is no enanthem seen in the course of the disease

A

There is no enanthem seen in the course of the disease

Scarlet fever is associated with bacterial pharyngitis caused by Group A beta-hemolytic streptococci (GAS). Typically, scarlet fever is associated with acute pharyngitis. As a result, fever, sore throat, pain with swallowing, and cervical adenopathy is present. If there is no pharyngitis, the source of infection can be a wound or burn which is infected with GAS. In addition to a classic sandpaper rash, physical findings can include a strawberry tongue (enanthem of the disease) and pastia lines. Pastia lines are pink or red lines formed of confluent petechiae, which are found in folds of the skin such as the antecubital fossa. The rash usually persists for about one week and may be followed by desquamation.

66
Q

A 16-year-old female presents for a one-year history of recurrent episodes of a pruritic skin rash. The skin lesions last for fewer than 24 hours and there is no residual scarring. She denies history of fever. She is otherwise healthy. On examination, there are pink to red papules and plaques with a central pallor of different sizes on her axilla, back and legs. Which of the following is the first line of treatment for this patient’s condition?

Oral antihistamine
Topical antihistamine
Oral steroid
Subcutaneous adrenaline
Topical steroid

A

Oral antihistamine

The patient in this clinical scenario presents with chronic urticaria.
The principles of management are to avoid exacerbating factors and to control symptoms as long as the urticaria persists. Pharmacological agents are directed at preventing mast cell degranulation and the effects of mast cell mediators released. H1- antihistamines (e.g., cetirizine, loratadine, fexofenadine), taken regularly, are the first-line pharmacological treatment. The dose can be up-titrated to 4 times the standard dose if symptoms remain at 2 to 4-week intervals.

67
Q

A 17-year-old girl was well until about 3 weeks ago, when she began complaining of being “tired all the time.”On examination, her temperature is 38°C . Her cervical CT showed a lesion on the cervical vertebrae.. Hemoglobin: 10.2; WBC: 9600 with increased neutrophils. She has needle injuries on her forearms. Gram-negative rods grew in the blood culture.

S.aureus S. typhi S.pyogenes E.faecalis P.aeruginosa

A

P.aeruginosa

Pseudomonas aeruginosa hematogenous osteomyelitis is often seen in intravenous drug abusers, and this organism has a predilection for the cervical vertebrae.

68
Q

A 19-year-old woman presents to the emergency department due to a sudden- onset rash. She reports that she presented one week ago, where she was diagnosed with streptococcal pharyngitis and prescribed amoxicillin. Currently, she complains of a sore throat and fatigue. The patient has no past medical history and takes no regular medications. On examination, she has an erythematous oropharynx without exudates, posterior cervical lymphadenopathy, splenomegaly, and a diffuse maculopapular rash without any mucosal involvement.
What is the most likely diagnosis?

Toxic epidermal necrolysis
Morbilliform drug eruption
Fixed drug eruption
Toxic shock syndrome
Stevens-Johnson syndrome

A

Morbilliform drug eruption

This patient with a sore throat, fatigue, posterior cervical lymphadenopathy, and splenomegaly most likely has infectious mononucleosis. Patients who have mononucleosis and take a penicillin antibiotic, such as amoxicillin, can develop a morbilliform drug eruption. This patient is most likely having an adverse drug reaction (ADR). The most common drugs causing cutaneous ADRs are penicillin derivatives, allopurinol, sulfonamides, and nonsteroidal anti- inflammatories (NSAIDs). Mucous membrane involvement is rare. It would indicate a severe cutaneous ADR such as Stevens-Johnson syndrome, toxic epidermal necrolysis, or a drug hypersensitivity reaction.

69
Q

A 56-year-old woman presented with a 2 months history of pruritic papules on the wrists. Dermatological examination showed numerous violaceous, polygonal, shiny papules with reticular white lines on their surfaces. In addition, bilateral buccal mucosa showed reticular white lines in a lace-like pattern.
What is your most likely diagnosis in this patient?

Discoid eczema
Allergic contact dermatitis
Lichen planus
Pityriasis rosea
Guttate psoriasis

A

Lichen planus

Lichen planus (LP) is a pruritic, papular eruption characterized by its violaceous color, shiny appearence, polygonal shape, and fine scale. It is most commonly found on the flexor surfaces of the upper extremities. It can occur at any age but two thirds of patients are aged 30-60 years. White lines, called Wickham Stria, are often found on the papules. Pruritus is common and severe in LP. Oral lesions are classified as reticular, plaque-like, atrophic, papular, erosive, and bullous. A lace-like pattern on the buccal mucosa and the tongue is referred to as the reticular form of LP.

70
Q

A 68-year-old male patient presents to the dermatologist with a skin lesion on the dorsum of his left hand. The lesion is a red papulo-nodule with scale and ulceration. A biopsy is undertaken, and a diagnosis of cutaneous squamous cell carcinoma is made.
What would most likely be found on histological examination of the biopsy?

Proliferation of basaloid cells forming nests
Atypical melanocytes
Central invagination with a keratotic core
Keratin pearls
Keratin filled multiple cysts

A

Keratin pearls.

Keratin pearls’ are characteristically found in cutaneous squamous cell carcinomas.
Atypical melanocytes are found in melanomas, not squamous cell carcinoma.
Central invagination with a keratotic core is more consistent with a diagnosis of a keratoacanthoma.
Proliferation of basaloid cells forming nests describes the histopathological finding in a basal cell carcinoma.
Keratin-filed cysts are the classic histological finding in seborrheic keratosis, which is a benign proliferation of immature keratinocytes and is classically described as a well demarcated lesion with a ‘stuck-on’ appearance.

71
Q

Where does the sacro il iac jo int pa in rad iate to?

Anter or crur s
Ventral part of th gh
Lateral part of crur s
Sole of foot
Gluteal reg on and poster or th

A

Gluteal region and posterior thigh

72
Q

Where does the sacro il iac jo int pa in rad iate to?

Anter or crur s
Ventral part of th gh
Lateral part of crur s
Sole of foot
Gluteal reg on and poster or th

A

Gluteal region and posterior thigh

73
Q

Trendelenburg ga it is caused by the fa ilure of wh ich muscle?

M. sartor ius
M. gluteus max imus
M. extensor halluc us longus
M. gluteus med ius
M. t ib al is anter ior

A

Gluteus medius

74
Q

A 70-year-old male patient notices that a lesion that has been on the dorsum of his right hand for 2-3 years, has recently changed shape and has bleeding areas. The lesion is macroscopically skin-colored, hyperkeratotic and contains milimetric bleeding spots. In the biopsy taken from the lesion, both precursor areas and invasive focus suspected of squamous cell carcinoma were detected.
Which of the following lesion is most likely the precursor lesion described above?

Fibroepithelial polyp
Actinic keratosis
Seborrheic keratosis
Epithelial cyst

A

Actinic keratosis

Actinic keratosis is an epidermal dysplasia developing in skin sites commonly exposed to sunlight such as the face, ears, dorsum of the hands.They can be s kin colored, red or tan lesions usually smaller than 1 cm in diameter. It is considered as preneoplastic lesion of the skin. When the lesions are left untreated, may transform to squamous cell carcinoma.

75
Q

Related with necrotising fasciitis infections, which one is the most important factor associated with higher mortality rates?

Delay to first debridment
Greater degree of organ dysfunction
Renal failure
Greater extend of disease
Disease due to Group A Streptococci

A

Delay to first debridment

76
Q

Which of the following defined histological levels would be appropriate to use when measuring the “Breslow thickness” in a patient with malignant melanoma?

Stratum basale- Most superficial part of the tumor Stratum basale- Deepest part of the tumor
Stratum granulosum- Most superficial part of the tumor
Stratum corneum- Most superficial part of the tumor
Stratum granulosum- Deepest part of the tumor
Stratum corneum- Deepest part of the tumor

A

Stratum granulosum- Deepest part of the tumor

Breslow thickness is expressed in millimeters and measures depth from the granular layer of the epidermis to the deepest part of the tumor •Breslow thickness is strongly correlated with melanoma survival and is a component of the clinical staging system for melanoma

77
Q

Which of the following findings are true for atopic dermatitis (AD)?

I-AD usually starts on the cheeks and extensor extremities in infancy
II- Diaper area is usually affected in infants with AD
III- Typical involvement areas are the extensor areas in childhood AD
IV- Presence of personal or family history of atopy is one of the major criteria for the diagnosis of AD

A

1 4

Typical involvement areas are the flexural areas, such as antecubital Feedback:
fossae and popliteal fossae, in chilhood. In contrast to seborrhoeic dermatitis, diaper area is usually spared in infants with AD.

78
Q

Which of the following is incorrect regarding bacterial and viral Infections in skin and soft tissue?

The very severe form of deep tissue infection called necrotizing fasciitis is often caused by Pseudomonas aeruginosa.

Diabetes mellitus results in progressive peripheral neuropathy and small vessel occlusion. These conditions lead to inadvertent trauma, poor wound healing, and tissue necrosis

Most of the superficial localized infections (impetigo, folliculitis, furuncles) are caused by S. aureus or beta-hemolytic streptococci

Cellulitis is an inflammatory process involving the skin and supporting tissues, with some extension into the subcutaneous tissues

Skin and soft tissue infections are common in every age group. They can infect the young and physically active as well as the elderly and sedentary.

A

The very severe form of deep tissue infection called necrotizing fasciitis is often caused by Streptococcus pyogenes.

79
Q

A 4-year-old girl was taken to the pediatrician by her mother due to bulging of left eye and vision problems.
After physical examination and radiological evaluation, a local mass affecting left eye was detected. In the
biopsy performed from the mass, malignant cells consisting of small round cells and tad-pole-like cells were
observed.

Which of the following is the most likely diagnosis based on current histopathological findings?

Rhabdomyosarcoma
Kaposi sarcoma
Liposarcoma
Schwannoma
Lymphangioma

A

Rhabdomyosarcoma

Malignant cells consisting of small round cells and tad-pole-like cells are features of
rhabdomyosarcomas as well as spindle cells and pleomorphic cells. Liposarcomas would
reveal atypical adipocytes, Kaposi sarcoma would vascular structures. Schwannoma and
lymphangioma are benign tumors.

80
Q

A 45-year-old female presents to the rheumatology clinic with complaints of joint pain
and stiffness, primarily in the small joints of the hands and feet. She reports that the
symptoms have been progressively worsening over the past several months and are
particularly prominent in the mornings. Upon further questioning, she mentions fatigue.
Physical examination reveals symmetric swelling and tenderness in the proximal
interphalangeal joints and metacarpophalangeal joints, along with limited range of
motion. No significant findings on other systems. Additionally, she has mild ulnar
deviation of the fingers and swan-neck deformity of the digits.
Before the laboratory investigations; which of the following is the most likely diagnosis
of this patient?

Osteoarthritis
Rheumatoid arthritis
Psoriatic arthritis
Gout
Systemic lupus erythematosus

A

Rheumatoid arthritis

Info: Rheumatoid arthritis (RA) is a chronic autoimmune disease
characterized by symmetrical polyarthritis, typically involving the small
joints of the hands and feet. Patients often experience morning
stiffness, joint swelling, and fatigue. Laboratory tests may show
elevated acute-phase reactants (ESR, CRP) and positive rheumatoid
factor (RF) and anti-cyclic citrullinated peptide (anti-CCP) antibodies.

81
Q

Match these with Osteoarthritis and Rheumatoid arthritis.

Formation of osteophytes [a]
Subchondral sclerosis [b]
Bouchard nodes [c]
Heberden nodes [d]
Decreased joint space [e]
Hypertrophy of joint capsule [f]
Formation of pannus [g]
Symmetrical involvement [h]

A

Formation of osteophytes –> Osteoarthritis
Subchondral sclerosis–> Osteoarthritis
Bouchard nodes–> Osteoarthritis
Heberden nodes–> Osteoarthritis
Decreased joint space–> Osteoarthritis
Hyperthrophy of joint capsule–> Osteoarthritis
Formation of pannus–> Rheumatoid Arthritis
Symmetrical involvement–> Rheumatoid Arthritis

82
Q

Match the following definitions regarding the bone fracture types with the appropriate
nomenclature of the fractures.

Name of the fracture when the overlying skin is not intact [a]

Name of the fracture when the bone is broken due to unknown prostate carcinoma [b]

Name of the fracture when the ends of the bone at the fracture site are aligned [c]

Name of the fracture when the bone is fragmented [d]

Name of the fracture after repetitive loads of the same bone [e]

Compound fracture
Pathologic fracture
Greenstick fracture
Stress fracture
Simple fracture
Comminuted fracture
Displaced fracture
Non-displaced fracture

A

Name of the fracture when the overlying skin is not intact
Compound fracture

Name of the fracture when the bone is broken due to unknown
prostate carcinoma Pathologic fracture

Name of the fracture when the ends of the bone at the fracture site
are aligned Non-displaced fracture

Name of the fracture when the bone is fragmented
Comminuted fracture

Name of the fracture after repetitive loads of the same bone
Stress fracture

83
Q

Bony callus formation [A]
Fibrocartilaginous matrix formation [B]
The bone composed of irregularly arranged collagen fibers [C]
The bone composed of parallel arranged collagen fibers [D]
The last step of inflammatory phase of fracture healing [E]
The second phase of fracture healing [F]

Inflammation
Reparative phase
Granulation tissue formation
Hematoma formation
Lamellar bone
Woven bone
Soft callus
Hard callus
Remodeling phase
Inflammatory phase

A

Bony callus formation Hard callus
Fibrocartilaginous matrix formation Soft callus
The bone composed of irregularly arranged collagen fibers
Woven bone
The bone composed of parallel arranged collagen fibers
Lamellar bone
The last step of inflammatory phase of fracture healing
Granulation tissue formation
The second phase of fracture healing Reparative phase

84
Q

What are the most common symptoms of musculoskeletal system disorders that are
associated with chronic diseases and also can diminish the quality of our lives?
(Choose as many as required)

Joint noises
Rash
Intentional weight loss
Muscle Soreness
Fatigue
Papules
Pain
Stiffness
Disability

A

Muscle Soreness
Fatigue
Pain
Stiffness
Disability

Info: pain is the main symptom of MSD muscle soreness, disability
fatigue stiffness are the others. painful noises is meaningful

85
Q

Which of the below listed tests aim to search pathologic reflexes? (Choose as many as required)

Laseque Test
Hoffmann Test
Ankle Clonus
Babinsky Test
Femoral Stretch Test

A

Hoffmann Test, Babinsky Test and Ankle Clonus

Hoffmann Test, Babinsky Test and Ankle Clonus are pathologic
reflexes, wheras Laseque Test and Femoral Stretch Test are
provocative tests for nerve root compression

86
Q

Which of the following features are considered as a risk factor in the development of
Developmental Dysplasia of the Hip (DDH)? (Choose as many as required)

Positive family history
Aged mother
Male sex
Vertex presentation at birth
Intrauterine packaging problems

A

Positive family history and intrauterine packaging problems.

Info: Firstborn (primiparity), Young mothers, Oligohydramnios, Intrauterine
packaging problems, Positive family history, Breech presentation at birth, Female sex, White skin color, Newborns with greater weight and height and Newborns with deformities of the feet or spine are considered factors for the development of DDH in humans. Male sex, Vertex presentation at birth and Aged mothers as listed here considered as risk factors for DDH.

87
Q

Which of the following is not correct for the Codman Triangle (periosteal reaction)?

Commonly seen in benign bone tumors
A radiographic feature of bone tumors
Suggestive of an aggressive or rapidly growing bone tumor such
as osteosarcoma
Occurs when rapidly growing bone tumors
Appears as a triangular area on the cortex

A

Commonly seen in benign bone tumors, this is incorrect.

Info: Codman triangle is a radiologic sign seen most commonly on
musculoskeletal plain films. It is the name given to a periosteal
reaction that occurs when bone lesions grow so aggressively that
they lift the periosteum off the bone and do not allow the periosteum
to lay down new bone.

88
Q

Which of the following is the definition of allodynia?

An increased sensitivity to a stimulus

Exaggerated pain response from a stimulus that normally elicits pain

Pain elicited from a nonpainful stimulus

An unpleasant sensation that is either evoked or spontaneous in nature

Diminished sensitivity to a normally painful stimulus

A

Pain elicited from a nonpainful stimulus

89
Q

Which of the following refers to the most common benign bone tumor consisted of a
cartilage-cap that is attached to the underlying skeleton by a bony stalk?

Osteochondroma
Chondrosarcoma
Osteoblastoma
Osteoid osteoma
Osteosarcoma

A

Osteochondroma

Osteoid osteoma and osteoblastoma shows similar features histological
not contain a cartilage cap. Osteosarcoma and chondrosarcoma are ma
tumors. Osteochondroma is a cartilage forming bone tumor that is also n
exostosis. It is the most common benign bone tumor consisted of a beni
cap that is attached to the underlying skeleton by a bony stalk. Most com
in the metaphysis of long bones (femur > humerus > tibia) and there is m
predominance

90
Q

Which of the following symptoms/signs is consistent with inflammatory arthritis?

Pain in the joint that occurs only with flexion
Swelling only on the anterior side of the joint
Joint pain that is worse in the evening
Morning stiffness lasting more than an hour
Joint pain that increases with activity

A

Morning stiffness lasting more than an hour

Inflammatory joint pain involves the joints diffusely and is present at
rest and with normal use. Nocturnal pain may interfere with sleep.
This joint pain is associated with prolonged stiffness for greater than
30 to 60 minutes, which is worse in the morning or after inactivity
(gelling). Fatigue is common, may be severe, and typically occurs by
early afternoon after stiffness has improved.

91
Q

Which of the following trauma mechanisms would highly cause shoulder dislocation
in an adult?

DOMS (delayed onset muscle Soreness)
FOOSH (fall on the outstreched hand)
Strain injury
Sprain injury
Force transmission from steering wheel in a traffic accident

A

Force transmission from steering wheel in a traffic accident

Info: force transmission vector may produce shoulder dislocation in a
traffic accident. dashboard also may cause a hip dislocation or
fractures

92
Q

Which of the listed joints are considered as a fibrous joint? (Choose as many as
required)

Symphisis pubis
Proximal radio-ulnar joint
Intervertebral discs
Cranial sutures
Distal tibio fibular joint

A

Cranial sutures
Distal tibio fibular joint

Info: Distal tibio fibular joint and cranial sutures are classified as fibrous
joints. Intervertebral discs and symphisis pubis are cartilaginous
joints, and proximal radio-ulnar joint is a synovial joint.

93
Q

Which one can not be seen in the Musculoskeletal trauma patient?

Hyperthermia

Consciousness

Hypotension

Hypoxia

Bleedings

Hypovolemia

A

Hyperthermia

Response
Feedback: Hyperthermia, frequently seen in patients following traumatic brain
injury (TBI), may be due to posttraumatic cerebral inflammation,
direct hypothalamic damage, or secondary infection resulting in
fever.

94
Q

Which of the following statements include correct information about bone and soft
tissue sarcomas seen in children and adolescents? (Choose as many as required)

A specific chromosomal translocation, t(11;22) (q24;q12) or a variant is
found in most of the Ewing sarcoma family of tumors.

Rhabdomyosarcoma (RMS) , the most common soft tissue sarcoma in
children, and may occur at virtually any anatomic site but are most
commonly at head and neck and genitourinary tract

Osteosarcoma is often located at the epiphysis or metaphysis of long
bones that are associated with maximum growth velocity with distal
femur, proximal tibia, proximal humerus being the most common sites of
origin.

Ewing sarcoma may originate from either long tubular bones or flat bones
extremities and the central axis (pelvis, spine, and chest) , femur and pel
most common sites.

Osteosarcoma occurs equally at any age starting form infancy to elde
originates from either long tubuler or flat bones

A

All are true except the last one.

Additional info: Osteosarcoma is often located at the epiphysis or metaphysis of long
are associated with maximum growth velocity (distal femur, proximal t
humerus), but any bone may be involved.
*Osteosarcoma most commonly affects adolescents; the peak inciden
during the period of maximum growth velocity, but may occur at any a
*The highest risk period for development of osteosarcoma is during th
growth spurt, suggesting an association between rapid bone growth a
transformation.

95
Q

A child is referred to emergency department with signs and symptoms of septic arthritis. What is the most likely source of bacteria?

Penetrating damage by accident
Hematogeneous route
Diagnostic puncture of the joint
Dissemination from osteomyelitis
Spread from an adjacent soft tissue infection

A

Hematogeneous route

96
Q

Which are following two types of receptors can be activated to reduce spasticity?

Alpha-1 adrenoceptors and the GABA receptors
The serotonin receptors and the GABA receptors
The alpha-2 adrenoceptors and the GABA receptors
The histamine receptors and the cholinergic receptors

A

The alpha-2 adrenoceptors and the GABA receptors

Info: The two types of receptors that can be manipulated to reduce
spasticity are the GABA receptors (by baclofen, diazepam) and the
alpha-2 adrenergic receptors (by tizanidine).

97
Q

Which of the following disease-modifying antirheumatic drugs (DMARDs) is a
mouse/human chimeric monoclonal antibody against TNF alpha?

Infliximab
Etanercept
Abatacept
Adalimumab

A

Infliximab

98
Q

Which of the following drugs might have altered serum and urine urate levels from
18 mg/dL and 800 mg/24 h to 7.2 mg/dL and 530 mg/24 h, respectively, in a 50-yearold woman in three weeks of therapy?

Naproxen
Furosemide
Acetylsalicylic acid
Indomethacin
Probenecid
Allopurinol

A

Allopurinol

Feedback:
Because the patient’s urate levels are decreased after therapy both
in serum and in urine, the drug must have decreased the formation of
uric acid. Allopurinol inhibits the conversion of hypoxanthine to
xanthine and xanthine to uric acid, thus decreasing uric acid
formation.

99
Q

Which of the following imaging findings is important in the diagnosis of calcium
pyrophosphate dihydrate arthritis?

Tophi
Chondrocalcinosis
Marginal erosion
Epin calcanei
Periarticular osteopenia

A

Chondrocalcinosis

Info: CPPD almost always occurs in joint tissues, most often in
fibrocartilage and hyaline cartilage and commonly leads to
chondrocalcinosis.

100
Q

A 5-year-old girl was admitted to the allergy clinic because of a rash. Parents first
noticed a red macular rash over her cheeks, and thought it was an allergic reaction to
the antibiotic prescribed 7 days ago, when a doctor saw her because of fever, cough
and malaise. However, 3 days after this red fascial rash, they noticed a lace-like redpink rash over her body. They noticed that the rash was recurring and vanishing
intermittently, for example when taking a warm bath, it was recurring, and when she
was playing outdoors, it was vanishing.
Physical examination was only remarkable for “ slapped cheek” appearance as well
as a red reticulate macular /urticarial exanthema (lace-like) all over the body. She
was otherwise healthy.
What is the most likely diagnosis ?

Erythema infectiosum
Chickenpox
Measles
Rubella

A

Erythema infectiosum

Info:
Upper respiratory infection findings followed by “Slapped cheek”
appearance and then apperance red reticulate macular /urticarial
exanthema (

101
Q

A 50-year-old female presents to the dermatologist for her yealy control. She reports a chronic skin condition characterized by red, scaly plaques on her elbows, knees, and scalp. Upon examination, you note the presence of well-demarcated plaques
covered with silvery-white scales. When the scales are removed, petechial
hemorrhages develop at the sites.
Which of the following is the most likely diagnosis of this patient?

Pemphigus vulgaris
Psoriasis
Lichen planus
Bullous pemphigoid
Discoid Lupus Erythematosus

A

Psoriasis

Info: Psoriasis is a chronic inflammatory skin condition characterized by wel
erythematous plaques covered with silvery-white scales.
Pemphigus vulgaris is a rare autoimmune disorder characterized by th
intraepithelial blisters and erosions on the skin and mucous membrane
presents with fragile blisters that rupture easily, leaving behind erosion
Bullous pemphigoid is another autoimmune blistering disorder characte
tense subepidermal blisters, often with erythematous or urticarial plaqu
pemphigus vulgaris, bullous pemphigoid lesions tend to be more locali
fragile.
Lichen planus is an inflammatory skin condition characterized by prurit
flat-topped papules and plaques with a violaceous hue. Wickham striae
lines or dots on the surface of the lesions, are a characteristic feature.
Discoid lupus erythematosus is a chronic autoimmune disorder characterised by
erythematous, scaly plaques with adherent scale and follicular pluggin
typically occur on sun-exposed areas such as the face, scalp, and ears
lead to scarring and dyspigmentation.

102
Q

A 55-year-old male presents with a rapidly growing, dome-shaped lesion on his right
forearm. He reports that the lesion appeared as a small, firm bump about a month
ago and has since grown rapidly in size. On examination, you note a well-defined,
erythematous papule with a central crateriform ulceration and a keratotic plug. There
is no surrounding erythema or induration. A biopsy of the lesion reveals a central cupshaped invagination filled with keratin and a central keratin plug.
Which of the following skin neoplastic diseases is most likely represented by the
clinical and histopathological findings described above?

Malignant Melanoma
Keratoacanthoma
Squamous Cell Carcinoma
Actinic Keratosis
Basal Cell Carcinoma

A

Keratoacanthoma

Info: Keratoacanthomas are dome-shaped nodules with a central
keratin plug, often appearing rapidly and spontaneously
regressing.
Basal cell carcinomas typically present as pearly or translucent
nodules with telangiectatic vessels and rolled borders.
Squamous cell carcinomas often appear as hyperkeratotic,
erythematous plaques or nodules with central ulceration.
Melanomas typically present as asymmetric, pigmented lesions wit
borders and color variation.
Actinic keratoses are typically erythematous, scaly patches or plaq
found in sun-exposed areas.

103
Q

A 57-year-old female presents to the hospital with a worsening, diffuse, bullous
eruption. The eruption started four weeks prior and was distributed mainly on her
lower extremities. The patient went to her primary care physician, who prescribed
antibiotics and told the patient that she had cellulitis. The patient took the antibiotics,
but the rash continued to worsen. After completing the antibiotic course without
improvement, the patient presented with diffuse and erythematous tense bullae
ranging from 1.5 to 2 centimeters in diameter. The lesions can be appreciated on the
patient’s face, neck, back, chest, abdomen, and extremities. The lesions were both
pruritic and painful. On examination, the patient was afebrile and without
leukocytosis, yet C-reactive protein was elevated. A punch biopsy performed showed
a subepidermal blister with eosinophils and neutrophils. The underlying dermis
demonstrated severe edema and infiltrate of eosinophils and lymphocytes. Direct
immunofluorescence (DIF) of the skin revealed the linear deposition of
immunoglobulin G (IgG) and complement C3 along the dermo-epidermal junction.
Based on the clinical presentation, histopathologic features, and DIF findings, which
of the following is the most likely diagnosis?

Dermatitis herpetiformis
Hailey hailey diasease
Bullous pemphigoid
Pemphigus vulgaris
Linear IgA bullous dermatosis

A

Bullous pemphigoid

Bullous pemphigoid is the most common of the blistering disorders,
characterized by tense bullae or blistering lesions filled with clear
fluid. Histologically, it presents with a subepidermal blister with
eosinophils and neutrophils. Direct immunofluorescence typically
reveals linear deposition of IgG and complement C3 along the dermoepidermal junction.

104
Q

A tissue scraping sample of a 5 year old girl with an unexplained atypical vesicular
rash revealed multinucleated giant cells in a Tzanck test.
Which of the following statement is definitely correct?

It is a herpes simplex 2 virus infection
It is a herpes simplex 1 virus infection
It is a varicella zoster virus infection
It can be a herpes simplex or varicella zoster virus infection
It is a herpes zoster infection

A

It can be a herpes simplex or varicella zoster virus infection

Info: Vesicular rash sample Tzank test revealing multinucleated giant
cells can be caused by HSV 1, HSV 2 or VZV. The definitely true
answer is : It can be a herpes simplex (HSV1 and 2) or varicella
zoster virus infection
A definitely true answer is HSV 1, HSV 2 or VZV. The others lack
the other responsible viruses.

105
Q

Candidal infection of the mouth: [A]
Dermatophyte infection of nail plate: [B]
Candidal infection of the glans of the penis: [C]
Candidal infection of skin fold: [D]
Superficial infection of inguinal area: [E]
Superficial fungal infection of body surfaces: [F]
Superficial fungal infection of the foot [G]
Superficial fungal infection of the beard area [H]

Balanitis
Onychomycosis
Thrush
Tinea cruris
Tinea pedis
Intertrigo
Tinea barbae
Tinea inguinalis

A

Candidal infection of the mouth: Thrush
Dermatophyte infection of nail plate: Onychomycosis
Candidal infection of the glans of the penis: Balanitis
Candidal infection of skin fold: Intertrigo
Superficial infection of inguinal area: Tinea inguinalis
Superficial fungal infection of body surfaces: Tinea cruris
Superficial fungal infection of the foot Tinea pedis
Superficial fungal infection of the beard area Tinea barbae

106
Q

Circumscribed flat lesion larger than 10 mm in diameter [A]
Elevated flat-topped lesion smaller than 10 mm [B]
Elevated roughly spherical lesion greater than 10 mm [C]
Discrete, pus filled raised lesion [D]
Fluid filled raised lesion less than 10 mm [E]
Intercellular edema of the epidermis [F]
Intracellular edema of the epidermis [G]
Complete loss of epidermis and a part of dermis [H]
Loss of cohesion between keratinocytes [I]
Diffuse epidermal hyperplasia [J]
Thickening of the stratum corneum [K]

Bulla
Vesicle
Acantholysis
Acanthosis
Nodule
Macule
Erosion
Plaque
Patch
Spongiosis
Hydropic swelling
Papule
Hyperkeratosis
Ulceration
Pustule

A

Circumscribed flat lesion larger than 10 mm in diameter Patch
Elevated flat-topped lesion smaller than 10 mm Papule
Elevated roughly spherical lesion greater than 10 mm Nodule
Discrete, pus filled raised lesion Pustule
Fluid filled raised lesion less than 10 mm Vesicle
Intercellular edema of the epidermis Spongiosis
Intracellular edema of the epidermis Hydropic swelling
Complete loss of epidermis and a part of dermis Ulceration
Loss of cohesion between keratinocytes Acantholysis
Diffuse epidermal hyperplasia Acanthosis
Thickening of the stratum corneum Hyperkeratosis

107
Q

Which of the following choices are correct in terms of the characteristics of the rash
(eruption) seen in viral diseases in children? (Choose as many as required)

The most important characteristics of rubella rash is being in a central
distribution, umbilication, and presence of different highly pruritic skin
rashes in the same anatomic area
Roseola infantum is caused by HHV 6 and 7 , and the first clinical
symptom is abrupt onset of high fever (40°C), which lasts for 3 days with
nonspecific complaints followed by an abrupt loss of fever and the onset
of a mild, pink, morbilliform rash
The typical measles rash begins to erupt behind the ears and in the
hairline area, then spreads over the rest of the skin over a period of a
few days, and typically resolves in the same order as its appearance,
and will often desquamate
Rubella eruption appears on the face first, spreading in a cephalocaudal
trend over 1 - 3 days, and tends to fade in 2 - 3 days in the same
manner as it appeared without desquamation

A

All except the first one are true.

The first one would be correct if it was varicella instead of rubella.

108
Q

Which of the following clinical findings about a melanocytic lesion that has been
followed for 3 months raise suspicion for malignant melanoma or dysplastic nevus ?
(Choose as many as required)

Both halves of the lesion are similar and match each other
Itchy for last 3 weeks
Mixed color in the same lesion
Same size since the first day
The edges of the lesion is irregular
5 mm in diameter

A

Itchy for last 3 weeks
Mixed color in the same lesion
The edges of the lesion is irregular

Info: Clinical evaluation of melanocytic lesions (When to suspect of a
malignancy?):
Asymmetry: Half of the lesion does not match the other half
Border irregularity: The edges are ragged, notched, or blurred
Color variegation: Pigmentation is not uniform and may display
shades of tan, brown, or black; white, reddish, or blue discoloration
is of particular concern
Diameter: A diameter greater than 6 mm is characteristic, although
some melanomas may be smaller in size; any growth in a nevus
warrants an evaluation
Evolving: Changes in the lesion over time are characteristic; this
factor is critical for nodular or amelanotic (nonpigmented)
melanoma, which may not exhibit the ABCD criteria above

109
Q

Which of the following features regarding Dermatitis herpetiformis
are correct? (Choose as many as required)

Ig A deposition on direct immunofluorescence occurs in a continuou
linear pattern at the dermoepidermal junction

The skin lesions are extremely itchy

Histopathologically the detachement is at the dermoepidermal
junction

A chronic vesiculobullous skin disorder characterized by
autoantibodies against desmoglein

It is one of the extraintestinal manifestation of ulcerative colitis

A

The skin lesions are extremely itchy
Histopathologically the detachement is at the dermoepidermal
junction

Info: A chronic vesiculobullous skin disorder characterized by autoantib
tTG2
Ig A deposition on direct immunofluorescence occurs in a granula
the dermoepidermal junction
Histopathologically the detachement is at the dermoepidermal jun
The skin lesions are extremely itchy
It is the specific cutaneous manifestation of celiac disease

110
Q

Which of the test below is not helpfull in dermatophytoses diagnosis?

Microscopic examination of roots of hair

Wood lamb examination of the affected skin lesions

Culture of skin and nail scraping samples

Serology

Skin scraping sample examination with15- 20% KOH preparation

A

Serology

ınfo: In dermatophyte infections serology is not helpful in microbiological
diagnosis
Microscopic examination of roots of hair, skin scraping sample
examination with15-20% KOH preparation, culture of skin and nail
scraping samples and Wood lamb examination of the affected skin
can be used in the diagnosis of dermatophyte infections.

111
Q

A 70-year-old male presents to the dermatology clinic with a new skin lesion on his che
examination, you observe a dark brown, slightly raised lesion with smooth borders (Fig
excisional biopsy is performed, and microscopic examination reveals hyperkeratosis, k
horn cysts, and invagination cysts within the lesion.
Based on the clinical and histopathological findings, which of the following is the most
likely diagnosis of this lesion?

Seborrheic keratosis
Fibroepithelial polyp
Squamous cell carcinoma
Basal cell carcinoma
Malignant melanoma

A

Seborrheic keratosis

Info: Seborrheic keratosis is a common benign epidermal tumor characterize
circumscribed, tan to brown, waxy plaques or papules with a stuck-on a
Histologically, seborrheic keratosis shows hyperkeratosis, papillomatosi
cysts within the epidermis, along with basaloid cells and melanin pigme

112
Q

A 62-year-old man presents with painful oral lesions and skin blisters that started a week ago.
Physical examination reveals multiple flaccid (soft) bullae on the oral mucosa, chest, a
well as erosions and crusting over the lips (fig 1). Biopsy of a skin lesion shows intraep
formation with acantholysis (fig 2), and direct immunofluorescence reveals IgG deposit
epidermis (fig 3).
Which of the following is the most likely diagnosis of this patient?

Linear IgA bullous disease
Bullous pemphigoid
Pemphigus vulgaris
Dermatitis herpetiformis
Hailey-Hailey disease

A

Pemphigus vulgaris

113
Q

A 25-year-old woman presents to the emergency department with complaints of
sudden-onset, intensely itchy raised lesions on her arms, legs, and trunk. She reports
that these lesions appeared approximately 30 minutes after she ate a meal
containing shrimp, to which she has no idea about her allergy before. She denies any
difficulty breathing or swallowing. On examination, you observe multiple
erythematous, edematous, raised wheals distributed symmetrically on her body
(Figure). Some of the lesions have coalesced to form larger areas of swelling. A skin
biopsy is performed, which reveals superficial perivascular inflammation with dermal
edema and a sparse infiltrate of eosinophils in the dermis.
Based on the patient’s history, clinical presentation, and histopathologic features,
which of the following is the most likely diagnosis?

Acute eczematous dermatitis
Seborrheic dermatitis
Psoriasis
Bullous pemphigoid
Urticaria

A

Urticaria

Info: Urticaria, also known as hives, is a skin condition characterized by
the sudden onset of raised, erythematous, edematous wheals that
are intensely pruritic. It is often triggered by allergens, medications,
infections, or physical stimuli.
Acute eczematous dermatitis, also known as acute contact
dermatitis, is a type of eczema characterized by erythema, edema,
vesicles, and weeping or crusting lesions. It typically develops rapidly
after exposure to an irritant or allergen. Histop

114
Q

A 45-year-old woman presents to her dermatologist with complaints of an itchy rash
on her wrists and ankles that has been present for the past few weeks. Upon further
questioning, she reports a history of a similar oral mucosal white lesions a few
months ago. She denies any recent changes in medications or skincare products. On
examination, you observe flat-topped, polygonal, shiny papules with a violaceous
color distributed symmetrically on the flexor surfaces of her wrists and ankles. Some
lesions have a fine, lacy network of white lines overlying them.
Based on the patient’s history and clinical presentation, which of the following is
the most likely diagnosis?

Stevens Johnson Syndrome
Psoriasis
Lichen planus
Erythema multiforme
Toxic epidermal necrolysis

A

Lichen planus

Info: Lichen planus is an inflammatory skin condition characterized by
pruritic, polygonal, flat-topped papules and plaques with a violaceous
color. Wickham striae, fine white lines or dots on the surface of the
lesions, are a characteristic feature.

115
Q

A 24-year-old man comes to the office because of a 2-day history of a red, itchy rash
on his buttocks and legs.
Physical examination shows the findings in the photograph. The infectious agent
causing these findings most likely began to proliferate in which of the following
locations?

Eccrine gland
Dermis
Apocrine gland
Sebaceous gland
Hair follicle

A

Hair follicle

116
Q

A 29-year-old woman presents with an itchy and painful lesion of sudden onset on
her elbow, which has occurred in exactly the same location multiple times (Figure).
She denies recent trauma to the area and has had no direct contact with chemicals
or other agents, such as plants or corrosive substances. She reports no systemic
symptoms. Further questioning revealed that the lesion occurred recurrently and was
associated with menstruation, when she would take oral naproxen for menstrual
cramps.
What is the most likely diagnosis?

Bullous impetigo
Fixed drug eruption
Herpes simplex virus infection
Erythema multiforme
Bullous insect bite reaction
Question 47
Which of the below histopathological findings is not specific for psoriasis?
Selected Answer: Hypergranulosis
Answers: Hypergranulosis
Microabscesses
Parakeratosis
Lymphocytic infiltration in the dermis
Acanthosis
Response
Feedback:
One of the main features of psoriasis histopathology is
“Hypogranulosis” which describes the thinning of the granular layer.
Hypogranulosis is typically seen in diseases of increased cell
turnover such as psoriasis.
Question 48
Which of the following agents used in the treatment of severe cases of psoriasis may
be combined with disease-modifying antirheumatic drugs (DMARDs)?
Selected Answer: Glucocorticoids
Answers: Vitamin D analogues
Glucocorticoids
Calcineurin inhibitors
Retinoids
Response
Feedback:
Analogs of vitamin D3 increase differentiation and inhibit proliferation
of keratinocytes and therefore used both as monotherapy and in
combination with topical corticosteroids, phototherapy (psoralen–
ultraviolet A [PUVA] or ultraviolet B [UVB]), acitretin (second
generation retinoid), methotrexate and cyclosporine.
Question 49
Which of the following statement regarding urticaria is correct?
Selected
Answer: Urticaria can co-exist with angioedema which is a deeper swelling
within the skin or mucous membranes
Answers: The first step tretment is first-generation H1 antihistamines
Dermographism is the most common form of acute urticaria. It is als
called dermatographia and dermographic urticaria

A

Fixed drug eruption

117
Q

Which of the below histopathological findings is not specific for psoriasis?

Hypergranulosis
Microabscesses
Parakeratosis
Lymphocytic infiltration in the dermis
Acanthosis

A

Hypergranulosis

Info: One of the main features of psoriasis histopathology is
“Hypogranulosis” which describes the thinning of the granular layer.
Hypogranulosis is typically seen in diseases of increased cell
turnover such as psoriasis.

118
Q

Which of the following agents used in the treatment of severe cases of psoriasis may
be combined with disease-modifying antirheumatic drugs (DMARDs)?

Vitamin D analogues
Glucocorticoids
Calcineurin inhibitors
Retinoids

A

Vitamin D analogues

Info: Analogs of vitamin D3 increase differentiation and inhibit proliferation
of keratinocytes and therefore used both as monotherapy and in
combination with topical corticosteroids, phototherapy (psoralen–
ultraviolet A [PUVA] or ultraviolet B [UVB]), acitretin (second
generation retinoid), methotrexate and cyclosporine

119
Q

Which of the following statement regarding urticaria is correct?

Urticaria can co-exist with angioedema which is a deeper swelling
within the skin or mucous membranes

The first step treatment is first-generation H1 antihistamines

Dermographism is the most common form of acute urticaria. It is als
called dermatographia and dermographic urticaria

Acute urticaria is urticaria, with or without angioedema, that is pres
4 weeks

A

Urticaria can co-exist with angioedema which is a deeper swelling
within the skin or mucous membranes

120
Q

In order to assess L3 root integrity, which function of the lower extremity muscles should be
observed?

Standing on his/her tip toes
Standing on his/her heels
Lifting his/her big toes o the ground
Squatting of the patient and rising from squatting
Flexing his/her hips

A

Squatting of the patient and rising from squatting

Feedback:
In order to assess L3 root integrity knee extensors should be tested. This could
be done by asking the patient to squat and rise from squatting

121
Q

In which of the groups below can Pseudomonas aeruginosa be a usually detected etiologic agent for septic
arthritis?

Neonates
Patients with Lyme disease
Sexually active adults
Intravenous drug user
People with prosthetic joints

A

Intravenous drug user

Info: Septic arthritis is generally caused by S. aureus and P. aureginosa in IV drug users. Other septic arthritis agents:

Neonates –> S. agalacticae
Kids and adults –> S. aureus ,
S. pyogenes, S. pneumoniae
Prosthetic joints (hip/knee) –> S. aureus, S. epidermidis
Sexually active adults –> N gonorrhea
Lyme disease –> Is caused by B. burdorferi, but it is an immune mediated condition and no mowill be in the
affected joint

122
Q

Septic arthritis symptoms may not include which of the following?

Fatigue
Weight loss
Chills
Relaxed position in the joint to increase joint space
Fever

A

Weight loss

123
Q

Which of the below listed contraction types denes elongation of the muscle due to the
resistance being greater than the force the muscle generates ?

Isotonic
Isometric
Eccentric
Co-concentric
Concentric

A

Eccentric

Eccentric contraction occurs where the lenght of the muscle increases
during contraction.

124
Q

Which of the following is a feature of neuropathic pain?

It can be related to the presence of chronic arthritis.

It can present with increased temperature, swelling and stiness.

It is a type of pain which is associated with the lesions of the pyramidal
tractus.

It can be associated with muscle weakness in the relevant area.

It is not associated with lesions of the somatosensory system.

A

It can be associated with muscle weakness in the relevant area.

Info: Neuropathic pain is a kind of pain arising from the lesions of the nervous
system, and can also present with weakness if a motor lesion is also involved.

125
Q

Which of the following is a spindle cell tumor morphologically?

Granular cell tumor
Embryonal rhabdomyosarcoma
Myxoma
Alveolar soft part sarcoma
Leiomyoma

A

Leiomyoma

Leiomyoma, solitary brous tumor, dermatobroma, brosarcoma,
leiomyosarcoma are among spindle shaped tumors morphologically. Glomus
tumor, embryonal and alveolar rhabdomyosarcomas, Ewing sarcoma,
neuroblastomas are among round cell tumors, granular cell sarcoma, alveolar soft
part sarcoma are among epithelioid cell tumors and myxomas are among myxoid
tumors.

126
Q

Which of the following statements are correct about rhabdomyosarcoma? (Choose as many as
required)

1) Represents an aggressive neoplasm

2) It has 4 histologic subtypes: alveolar, epithelial, palisading, spindle cell

3) It is a malignant mesenchymal tumor with skeletal muscle differentiation

4) Common locations include head and neck, genitourinary system, extremities
and retroperitoneum

5) Symptoms depend on the location and the size of the tumor

A

All except 2 are true.

There are 4 subtypes of tumor
histologically: alveolar, embryonal, pleomorphic, spindle cell

NOT epithelial or palisading.

127
Q

Which of the following symptoms/signs is consistent with noninammatory arthritis?

Pain not responding to NSAIDs
Systemic ndings
Joint pain that increases with activity
Morning stiness lasting more than an hour
Constitutional symptoms

A

Joint pain that increases with activity

Info: Patients with non-inammatory joint pain have pain with activity that is relieved
by rest. Although they may have stiness or gelling after inactivity, it typically
lasts less than 15 minutes, and systemic fatigue is not common.

128
Q

Which of the listed X ray ndings determine the abnormal position of femoral head relative to
the acetabulum in Developmental dysplasia of the hip (DDH)? (Choose as many as required)

Larger ossic nucleus of the proximal femoral physis (compared to the normal
side) in the acetabulum

Proximal femoral physis located at the inner lower quadrant

Proximal femoral physis resting above the Hilgenreiner line

High acetabular index (< 25°) at the aected side

Broken Shenton line at the aected side

A

3rd 4th and 5th statements are to be chosen.

Info: X ray ndins that should be sought for determination of the abnormal position
of femoral head relative to the acetabulum in DDH include:
1) Smaller ossic nucleus of the proximal femoral physis relative to the
contralateral side
2) Proximal femoral physis resting above the Hilgenreiner line
3 Proximal femoral physis not located at the inner lower quadrant
d) High acetabular index (< 25°) at the aected side
e) Broken Shenton line at the aected side

129
Q

A 50-year-old woman recently diagnosed with gouty arthritis started a treatment with a drug that
inhibits leukocyte migration and phagocytosis secondary to inhibition of tubulin polymerization.
Which of the following drugs did the patient most likely take?

Allopurinol
Colchicine
Indomethacin
Prednisone

A

Colchicine

Colchicine can be used to stop an acute gout attack or, most often, for the prevent
ion of further at tacks, as in this case. The drug binds to the intracellular protein,
tubulin, thereby preventing its polymerization into microtubules and thus blocking
mitosis in metaphase. Cells with the highest rate of division are aected early.
Granulocyte migration into the inamed area and phagocytosis of urate crystals by
macrophages are inhibited, thus relieving the pain and inammation of gouty
arthritis. These actions are specic, and the drug is devoid of general analgesic or
anti-inammatory eects.

130
Q

All of the following diseases except one of them are “mechanical disorders”
Which one is the exception?

Osgood Schlatters disease
Slipped capital femoral epiphysis
Perthes disease
Pes planus
Transient synovitis of hip

A

Transient synovitis of hip

131
Q

Which of the following disease-modifying antirheumatic drug (DMARD) would be most
appropriate for a 50-year-old patient recently diagnosed with mild rheumatoid arthritis?
He was a heavy smoker and had a history of chronic bronchitis with frequent acute
exacerbations usually treated with antibiotics. His joint pain continued despite ibuprofen
therapy

Rituximab
Hydroxychloroquine
Etanercept
Lefunomide

A

Hydroxychloroquine

The therapy of mild RA usually includes nonsteroidal antiinammatory drugs
(NSAIDs) that can control symptoms of pain and swelling, but DMARD treatment is
almost always added within 3 months of the RA diagnosis. Hydroxychloroquine
seems to be the least toxic among DMARDs and is usually the initial choice.
Immunosuppressants are relatively contraindicated in this patient because of his
frequent respiratory tract infections.

132
Q

Which of the following is not a screening test for all types of inflammatory arthritis?

Total Protein
C-Reactive Protein (CRP)
Erythrocyte Sedimentation Rate (ESR)
Anti-Cyclic Citrullinated Peptide (Anti-CCP)
Rheumatoid Factor (RF)

A

Total Protein

Info: 1.Screening blood tests for all types of inammatory arthritis:
–Erythrocyte Sedimentation Rate (ESR): An elevated ESR “supports”
the presence of an inammatory arthritis.
–C-Reactive Protein (CRP): This test is an alternative to obtaining the
ESR.
–Rheumatoid Factor (RF): A rheumatoid factor test should be
obtained when rheumatoid
arthritis (RA) is at least moderately possible in the patient.
–Anti-Cyclic Citrullinated Peptide (Anti-CCP): Measuring antibodies
to CCP is a new test for RA; it has higher specicity but lower
sensitivity than RF.
–Anti-Nuclear Antibody (ANA): ANA test is commonly obtained in
patients with arthralgias or
arthritis as a screening test for SLE or another connectivetissue disorder.
More than 95% of patients with SLE have ANA; thus, a negative ANA
result is a strong indicator that SLE is not present.
However, a positive ANA result lacks specicity and may occur in
persons with other connective-tissue diseases (Positive ANA
incidence is 99% in SLE, 85% in Sjögren’s, 88% in Scleroderma, 55%
in Rheumatoid Arthritis, and 40% in Juvenile Rheumatoid Arthritis)

133
Q

Which of the following is one of the common features of spondyloarthritis?

Raynaud’s phenomenon
Tophi
Enthesitis
Glomerulonephritis
Subcutaneous nodules

A

Enthesitis

Info: The most typical features of SpA are inammatory back pain, sacroiliitis,
spondylitis, asymmetric oligoarthritis and enthesitis. This group of diseases is
seronegative (autoantibodies seen in CTDs are absent). Extramusculoskeletal
ndings are anterior uveitis, psoriasis and IBD. SpA is strongly associated with
HLA-B27

134
Q

Which of the following is the mechanism of dantrolene administered to a 50-y.o. man who
developed muscle rigidity, tachycardia, labile blood pressure, profuse diaphoresis, and high fever
(>40°C) shortly after the initiation of general anesthesia with halothane and succinylcholine prior
to his inguinal hernia surgery? (The anesthesia was discontinued at once, and the drug of
interest was administered by rapid intravenous push.)

Blockade of Ca2+ channels in the sarcoplasmic reticulum

Blockade of Ca2+ channels in the skeletal muscle cell membrane

Increased K+ conductance in the skeletal muscle cell membrane

Blockade of excitatory neurotransmitter release in the brain

Activation of GABAB receptors in the spinal cord

A

Blockade of Ca2+ channels in the sarcoplasmic reticulum

Info: The clinical picture of the patient suggests the diagnosis of malignant
hyperthermia, a rare but potentially life-threatening disorder that can be triggered
by a variety of stimuli, including the administration of certain anesthetics (mainly
halothane) and neuromuscular blocking agents (mainly succinylcholine). The
disorder is related to a hereditary impairment in the ability of the sarcoplasmic
reticulum (SR) to sequester calcium (Ca2+); therefore, a prolonged release of Ca2+
occurs after the triggering event through SR Ca2+ channels (ryanodine receptors,
RyR). This causes massive muscle contraction, hyperthermia, and lactic acidosis.
Dantrolene is a drug of choice in this disorder because it blocks RyRs in the skeletal
muscle SR, thus preventing the massive release of Ca2+. Cardiac and smooth
muscle are minimally aected by dantrolene because they have a dierent subtype
of RyRs in their SR.

135
Q

A 19-year-old male presents to the dermatologist with complaints of intense itching, particularly
at the interdigital folds of his hands. He reports that the itching worsens at night. On physical
examination, elevated hyperemic rashes with evidence of scratching are noted. Some of the
lesions appear as thin, serpiginous lines. A biopsy of one of the lesions is performed. Microscopic
examination reveals a wedge-shaped inammatory inltrate within the dermis, composed of a
mixed inammatory cell population.
What is the most likely diagnosis for this patient?

Pemphigus vulgaris
Scabies
Tinea versicolor
Candidiasis
Molluscum contagiosum

A

Scabies

Info: The clinical and histopathological ndings are consistent with scabies. Scabies is
caused by the mite Sarcoptes scabiei, which burrows into the skin to lay eggs,
creating characteristic track-like lesions and causing intense pruritus, especially at
night.

136
Q

A 28-year-old woman presents to the dermatology clinic with multiple skin lesions on her hands.
She describes the lesions as raised, rough, and sometimes painful, especially when pressure is
applied. The lesions have been present for several months and have slowly increased in size and
number. On physical examination, you observe several well-circumscribed, rm, hyperkeratotic
papules with a rough surface. A biopsy of one of the lesions is performed. Histopathological
examination reveals epidermal hyperplasia, koilocytosis, keratohyalin granules, and
intracytoplasmic eosinophilic inclusions.
Based on the clinical presentation and histopathological ndings, what is the most likely
diagnosis?

Verrucae vulgaris
Impetigo
Molluscum contagiosum
Herpes infection
Tinea versicolor

A

Verrucae vulgaris

Info: The clinical and histopathological ndings are consistent with verrucae vulgaris,
also known as common warts. These lesions are caused by human papillomavirus
(HPV) infection and are characterized by epidermal hyperplasia, koilocytosis (a
change in epidermal cells due to viral infection), keratohyalin granules, and
intracytoplasmic eosinophilic inclusions.

137
Q

A 35-year-old woman presents to the clinic with complaints of severe itching and red, blistering
rashes on her hands and forearms. She reports that the symptoms started a few days after she
began using a new brand of dishwashing detergent. Physical examination reveals erythematous,
edematous plaques with oozing and crusting. A skin biopsy is performed, and histopathological
examination shows spongiosis with vesicle formation and a mixed inammatory inltrate in the
dermis.
Which of the following statements regarding “acute eczematous dermatitis” are correct? (Choose
as many as required)

Symptoms typically include intense itching and erythematous plaques
The condition is commonly triggered by external factors such as allergens or
irritants.
Histopathological ndings include spongiosis and dermal inammatory
inltrate.
Direct immunouorescence shows linear IgG deposition along the basement
membrane.
It is characterized by dermographism

A

The first three statements should be chosen.

Info: Acute eczematous dermatitis, also known as eczema, is an inammatory skin
condition commonly triggered by external factors such as allergens or irritants.
The histopathological hallmark is spongiosis (intercellular edema in the epidermis)
with vesicle formation and a mixed inammatory inltrate in the dermis.
Symptoms typically include intense itching and erythematous plaques.
Dermatographism, also known as dermographism urticaria, is an urticarial
eruption upon pressure or trauma to the skin.Additionally, direct
immunouorescence ndings in eczema are not characterized by linear IgG
deposition along the basement membrane, which is seen in conditions like bullous
pemphigoid.

138
Q

A 45-year-old woman presents to the dermatology clinic with painful blisters on her skin and in
her mouth. She rst noticed the blisters in her mouth two months ago, which then spread to her
chest and back. On examination, you observe multiple accid bullae and erosions on her chest,
back, and oral mucosa. A biopsy of a skin lesion is performed, and histopathological examination
reveals intraepithelial blister formation with acantholysis. Direct immunouorescence shows IgG
and C3 deposits in a sh-net pattern along the plasma membranes of keratinocytes.
Which of the following features regarding the patient’s condition are correct? (Choose as many
as required)

It can also involve mucosa as well as skin.
The blisters are mostly thin walled and painful.
It is the most common type of pemphigus.
IgG and C3 deposition occurs in a continuous linear pattern at the
dermoepidermal junction.
It is caused by autoantibodies that bind to proteins that are required for
adherence of basal keratinocytes to the basement membrane.

A

The first three statements are correct

139
Q

A 60-year-old woman presents with a slowly growing lesion on her left cheek. She rst noticed
the lesion several months ago, and it has gradually increased in size. On physical examination,
you observe a 1.5 cm pearly, translucent nodule with visible telangiectatic vessels and a slightly
rolled border. There is no ulceration or signicant surrounding erythema. The patient denies any
pain or itching associated with the lesion. A biopsy is performed, and histopathological
examination shows nests of basaloid cells with peripheral palisading and retraction artifacts
within the dermis.
Which of the following skin neoplastic diseases is most likely represented by the clinical and
histopathological ndings described above?

Keratoacanthoma
Squamous Cell Carcinoma
Malignant Melanoma
Basal Cell Carcinoma
Actinic Keratosis

A

Basal Cell Carcinoma

Info: The clinical presentation of a pearly, translucent nodule with telangiectatic
vessels and rolled borders, along with histopathological ndings of nests of
basaloid cells with peripheral palisading and retraction artifacts, is characteristic
of Basal Cell Carcinoma (BCC).
Squamous Cell Carcinoma: Typically presents as hyperkeratotic, erythematous
plaques or nodules with central ulceration.

Malignant Melanoma: Usually appears as asymmetric, pigmented lesions with
irregular borders and color variation.
Keratoacanthoma: Presents as dome-shaped nodules with a central keratin plug
and rapid growth.
Actinic Keratosis: Appears as erythematous, scaly patches or plaques, often
found in sun-exposed areas

140
Q

In which of the conditions below will Eikenella corrodens be responsible of the ≈ 30 % of the
infections?

In decubitus ulcers
After a human bite
After a cat bite
After a dog bite
Diabetic foot infection

A

After a human bite

141
Q

What will be the result if yeasts are stained with a Gram stain?

It will be Gram negative stained
It can not be stained with Gram, lactophenol cotton blue staining is needed
visualization
It will be Gram positive stained
It will be Gram labile stained
It can not be stained with Gram, acidfast stain is needed visualization

A

It will be Gram positive stained

142
Q

Which of the following statements dene the clinical characteristics of Erythema infectiosum (fifth disease) correctly? (Choose as many as required)

It causes arthritis in young children commonly when compared to older children
and adults.
It is a common childhood disease caused by Human Parvovirus B19 which is
transmitted via aerosolized respiratory secretions, contaminated blood, or the
placenta.
“Glove and sock syndrome” consisting of a purpuric eruption with painful edema
and numerous small conuent petechiae at hands and feet is another
manifestation of erythema infectiosum .
Typical rash is in the form of reticulate macular or urticarial exanthema (lace-like)
and it can wax and wane with environmental changes, emotional factors, etc for
weeks to months.

A

All except the first one are correct.

Info: *Arthropathy only occurs in approximately 10% of children
*In older children, adolescents arthritis is more common (60% of adults) ,
while skin rash is less prominent and distinctive

143
Q

Which of the following statements does NOT define clinical characteristics of measles infection ?

Cough , coryza and conjunctivitis are common prodromal symptoms.
The rash of measles is in the form of coalescing erythematous macules and
papules.
Koplik spots may be seen during the prodromal phase.
The typical rash begins to erupt behind the ears and in the hairline area, then
spreads over the rest of the skin over a period of a few days.
The eruption typically resolves following a caudal to cranial order, without
desquamation.

A

Cough , coryza and conjunctivitis are common prodromal symptoms.

144
Q

A 25-year-old man presents to the emergency department with a sudden onset of red, targetshaped lesions on his palms and legs. He reports that the lesions appeared a few days after he
started taking a new medication for his upper respiratory infection. On examination, there are
multiple round erythematous macules, papules, vesicles, and bullae, some of which have a
characteristic central clearing with surrounding rings (g1). A biopsy from one of the skin lesions
shows vesicle formation and cytotoxic T cells attacking basal epithelial cells.
According to your most likely diagnosis , which of the following statements are correct? (Choose
as many as required)

The characteristic lesion is a targetoid (target) lesion.
Oral mucosa can not be involved in this disease
It is a type IV hypersensitivity reaction.
It is typically triggered by infections or medications.
Histopathology reveals cytotoxic T cells attacking basal epithelial cells

A

All except the second statement is correct.

Info: Erythema multiforme (EM) is an acute inammatory dermatosis often triggered by
infections (such as herpes simplex virus) or medications. It is a type IV
hypersensitivity reaction mediated by cytotoxic T cells that attack basal epithelial
cells. The characteristic lesion is a targetoid (target) lesion, which appears as a
central zone of necrosis surrounded by concentric rings of erythema.
Histopathological examination shows vesicle formation and cytotoxic T cells
targeting basal epithelial cells. Unlike chronic dermatoses like psoriasis, EM does
not involve chronic plaque formation and thickened skin. Oral mucosa can also be
involved in this disease.

145
Q

What is the most appropriate next step in the evaluation of an acanthosis nigricans patient?

Upper and lower endoscopy
Abdominal ultrasound
Bone marrow biopsy
Skin patch testing
MRI of the brain

A

Upper and lower endoscopy

Info: The patient exhibits signs of acanthosis nigricans, which, in the absence of
diabetes or obesity, can be a paraneoplastic syndrome associated with
gastrointestinal malignancies, particularly gastric adenocarcinoma. Therefore, an
upper and lower endoscopy is warranted to investigate potential underlying
malignancies.
Bone marrow biopsy: Not the most appropriate next step as the primary concern
is to rule out gastrointestinal malignancies associated with acanthosis nigricans.
MRI of the brain: Not indicated based on the presented symptoms and ndings.
Abdominal ultrasound: While useful, it does not provide the comprehensive
evaluation needed to detect gastrointestinal malignancies.
Skin patch testing: This is not relevant for the diagnosis of acanthosis nigricans or
its associated conditions.

146
Q

A 21 year-old woman presented with a 3 weeks history of erythematous scaly macules on the
body. The patient was otherwise in good health. She described the eruption as starting as a
single macule and spreading over the next 2 weeks. On physical examination, in addition to a
large round-oval macule with a collarette of ne scale, there were many small macules oriented
with their long axes along cleavage lines resembling a pine tree.
What is your most likely diagnosis in this patient?

Pityriasis rosea
Lichen planus
Guttate psoriasis
Nummular dermatitis
Atopic dermatitis

A

Pityriasis rosea

Info: Pityriasis rosea is an acute mild inflammatory exanthem characterized
by the development of erythematous scaly macules on the trunk. It is
more common in children and young adults. It begins with a solitary
salmon-colored macule that heralds the eruption (herald patch), and
over a few days it becomes a patch with a collarette of fine scale. Within
the next 1-2 weeks, a generalized exanthem usually appears. Bilateral
and symmetric macules with a collarette scale oriented with their long
axes along cleavage lines resembling a pine tree are typical for pityriasis
rosea.

147
Q

A 28-year-old male presents to the our out-patient clinic because he has an intensely itchy rash of 10 weeks duration. The lesions are present on and o but do not last more than 24 hours before new lesions develop. He has otherwise been healthy. The rash consists of circumscribed, raised, blanching erythematous papules and plaques, some of which have a central pallor, located on neck, arms and back. The mucous membranes, palms, and soles are spared. The rest of the examination is normal. What is the most likely diagnosis in this patient?

Urticaria
Contact dermatitis
Psoriasis
Drug eruption
Seborrheic dermatitis

A

Urticaria

148
Q

After tending her children, a woman develops red facial macules. These lesions subsequently
become vesicles that rupture and form honey-colored crusted erosions.
What is the most likely diagnosis?

Varicella
Contact dermatitis
Tinea faciei
Impetigo
Acne rosacea

A

Impetigo

Impetigo is a pyoderma that is usually caused by staphylococci or
streptococci.
The initial lesions are vesicles, which subsequently form characteristic
yellow, “honey-colored” crusts.
Impetigo is a common infection of the supercial layers of the epidermis
that is highly contagious.
Once a lesion is present, self-inoculation to other sites is very common.

149
Q

Which of the following ndings are correct for atopic dermatitis (AD)? (Choose as many as required)

Presence of personal or family history of atopy is one of the minor criteria for
the diagnosis of AD
Diaper area is usually spared in infants with AD
Typical involvement areas are the exor areas in childhood AD
AD usually starts on the cheeks and exor extremities in infancy

A

All except the last one are correct.

Info: AD usually starts on the face (cheeks) cheeks or on the extensor extremities
which are the places that are easy to scratch and rub as they are crawling in
infancy. And, typical involvement areas are the exural areas, such as antecubital
fossae and popliteal fossae, in childhood.

150
Q

Which of the following pharmaceutical system is not used to deliver agents to treat
dermatological disorders?

Emulsions
Transdermal therapeutic system
Ointment
Cream

A

Transdermal therapeutic system

Info: Transdermal Therapeutic Systems (TTS) or transdermal patches are special
systemic drug delivery systems that are applied directly to the skin. They are not
used to treat dermatological problems. The active substance is absorbed by the
skin and distributed through the body via the bloodstream.

151
Q

A 35-year-old woman presents with pain, fever and swelling in her right leg. Examination reveals
well-demarcated erythema and spreading streaks of lymphedema.
Which of the following is the most likely diagnosis of this patient’s condition?

Erysipelas
Necrotising fasciitis
Myositis
Impetigo
Necrotising cellulitis

A

Erysipelas

Response
Feedback: This patient has clinical features of erysipelas, a type of supercial cellulitis
with dermal and supercial lymphatic involvement.
Erysipelas is a bacterial infection that leads to pain, edema, and welldemarcated erythema.